Example Problems on Asymptotics

ST793 Homework 4 on Asymptotics

5.5, 5.9, 5.11, 5.13, 5.24

5.5

Consider the simple linear regression setting

\[Y_i = \alpha + \beta x_i + e_i, \ i = 1, \dots ,n,\]

where the $x_i$ are known constants, and $e_1, \dots , e_i$ are iid with mean 0 and finite variance $\sigma^2$. After a little algebra, the least squares estimator has the following representation

\[\widehat \beta - \beta = \frac{ \sum_{i=1}^n (x_i - \overline{ x }) e_i }{ \sum_{i=1}^n(x_i - \overline{ x })^2 }.\]

Using that representation, prove that $\widehat \beta \stackrel{ \text{p}}{\rightarrow} \beta$ as $n \rightarrow \infty$ if $\sum_{i=1}^n (x_i - \overline{ x })^2 \rightarrow \infty$.

We can proceed by Chebychev’s inequality.

\[\begin{align} \lim_{n \rightarrow \infty} P\Big(|\widehat \beta - \beta| > \epsilon) & = \lim_{n \rightarrow \infty} P(\frac{ | \sum_{i=1}^n (x_i - \overline{ x }) e_i |}{ \sum_{i=1}^n (x_i - \overline{ x })^2} > \epsilon\Big) \\ & = \lim_{n \rightarrow \infty} P\Big(| \sum_{i=1}^n (x_i - \overline{ x }) e_i | > \sum_{i=1}^n (x_i - \overline{ x })^2 \epsilon\Big) \\ & \leq \lim_{n \rightarrow \infty} \frac{ \text{Var} \Big( \sum_{i=1}^n (x_i - \overline{ x })e_i \Big) }{ \epsilon^2 \Big( \sum_{i=1}^n (x_i - \overline{ x })^2 \Big)^2 } \\ & = \lim_{n \rightarrow \infty} \frac{ \sum_{i=1}^n (x_i - \overline{ x })^2 \sigma^2 }{ \epsilon^2 \Big( \sum_{i=1}^n (x_i - \overline{ x })^2 \Big)^2 } \\ & = \lim_{n \rightarrow \infty} \frac{ \sigma^2 \sum_{i=1}^n(x_i - \overline{ x })^2 }{ \epsilon^2 \Big( \sum_{i=1}^n (x_i - \overline{ x })^2 \Big)^2 } \\ & = \lim_{n \rightarrow \infty} \frac{ \sigma^2 }{ \epsilon^2 \Big( \sum_{i=1}^n (x_i - \overline{ x })^2 \Big) } \end{align}\]

We know that the denominator goes to infinity, so the limit goes to 0. Thus, $\widehat \beta \stackrel{ \text{p}}{\rightarrow}\beta$.

5.9

Let $X_1, \dots , X_n$ be iid from a distribution with mean $\mu$, variance $\sigma^2$ and finite central moments, $\mu_3$ and $\mu_4$. Consider $\widehat \theta = \overline{ X } / s_n$, a measure of “effect size” used in meta-analysis. Prove that $\widehat \theta \stackrel{ \text{p}}{\rightarrow} \theta = \mu / \sigma$ as $n \rightarrow \infty$.

By WLLN we know $\overline{ X } \stackrel{ \text{p}}{\rightarrow} \mu$. The book also shows that $s_n^2 \stackrel{ \text{p}}{\rightarrow} \sigma^2$. Thus, by CMT we know $\sqrt{ s_n^2 } = s_n \stackrel{ \text{p}}{\rightarrow} \sigma = \sqrt{ \sigma^2 }$. Thus, since $\overline{ X }$ and $s_n$ converge piecewise and are independent, we can apply the continuous mapping theorem again. Take $g(a,b) = a / b$. Then

\[g(\overline{ X }, s_n) = \frac{ \overline{ X } }{ s_n }\stackrel{ \text{p}}{\rightarrow} \frac{ \mu }{ \sigma } = g(\mu, \sigma).\]

5.11

In the continuation of Example 5.20 (p. 230) on 231, it follows that under the local alternative $\mu_i = \mu + d_i/\sqrt{ n }$, $(k-1) F = \sum_{i=1}^k n (\overline{ X }_i - \overline{ \overline{ X }})^2/s_p^2$ converges in distribution to a noncentral chi-squared distribution with $k-1$ degrees of freedom and noncentrality parameter $\lambda = d^T (I_k - k^{-1} 1_k 1_k^T)d/\sigma^2 = \sum_{i=1}^k(d_i - \overline{ d })^2 / \sigma^2$. Now, if the data in a one-way ANOVA setup such as Example 5.20 (p.230) are normally distributed with means $\mu_1 , \dots \mu_k$ and common variance $\sigma^2$, then the exact power of the ANOVA $F$ statistic is the probability that a noncentral $F$ distribution with $k-1$ and $k(n-1)$ degrees of freedom and noncentrality parameter $\lambda = \sum_{i=1}^{k}n(\mu_i - \overline{ \mu })^2 / \sigma^2$ is larger than a percentile of the central version f that $F$ distribution. In R, that would be 1-pf(qf(.95,k-1,k*(n-1)),k-1,k*(n-1),nc), where $\text{nc} = \lambda$ and the level is 0.05. Show that if $\mu_i = \overline{ \mu } + d_i / \sqrt{ n }$, then $\sum_{i=1}^k (d_i - \overline{ d })^2 / \sigma^2 = \sum_{i=1}^k n(\mu_i - \overline{ \mu })^2 / \sigma^2$, that is, the asymptotic chi-squared approximation uses the same noncentrality parameter. In R that would be 1-pchisq(qchisq(.95,k-1),2,nc). Compare the approximation for $k=3$ and $nc=10$ to the true power for $n=5,10,$ and $20$.

Notice $d_i = \sqrt{ n } (\mu_i - \overline{ \mu })$ and

\[\overline{ d } = \frac{ 1 }{ k } \sum_{i=1}^k d_{i} = \frac{ 1 }{ k } \sum_{i=1}^{k} \sqrt{ n } (\mu_i - \overline{ \mu }) = 0.\]

Thus,

\[\sum_{i=1}^k \frac{ (d_i - \overline{ d })^2 }{ \sigma^2 } = \frac{ (\sqrt{ n } (\mu_i - \overline{ \mu }) - 0)^2 }{ \sigma^2 } = \frac{ n( \mu_i - \overline{ \mu }) }{ \sigma^2 }.\]

Using the R code provided, we get

\[\begin{array}{ r c c c } n & 5 & 10 & 20 \\ \text{Exact} & 0.7015083 & 0.7673536 & 0.7933118 \\ \text{Approximate} & 0.8154214 & 0.8154214 & 0.8154214. \end{array}\]

Notice that as $n$ increases our exact calculation gets closer to our approximation.

5.13

Suppose that $\widehat \theta_1$ is $\text{AN}(\theta_0, A_1/n)$ as $n \rightarrow \infty$.

a.

What can we say about the consistency of $\hat \theta_1$? Why?

By Theorem 5.13, if $A_i / n \rightarrow 0$, then $\widehat \theta_1$ is consistent for $\theta_0$.

b.

If $\widehat \theta_1 - \widehat \theta_2 \stackrel{ \text{p}}{\rightarrow} 0$, what can we say about the consistency of $\widehat \theta_2$? Why?

Assuming $ \widehat \theta_1 \stackrel{ \text{p}}{\rightarrow}\theta_0$,

\[\begin{align} \widehat \theta_1 - \widehat \theta_2 & \stackrel{ \text{p}}{\rightarrow} 0 \\ \widehat \theta_2 - \widehat \theta_1 & \stackrel{ \text{p}}{\rightarrow} 0 & \text{CMT} \\ \widehat \theta_2 - \widehat \theta_1 + \widehat \theta_1 & \stackrel{ \text{p}}{\rightarrow} 0 + \theta_0 & \text{CMT} \\ \widehat \theta_2 & \stackrel{ \text{p}}{\rightarrow} \theta_0 \end{align}\]

c.

If $\sqrt{ n }(\widehat \theta_1 - \widehat \theta_2) \stackrel{ \text{p}}{\rightarrow} b $, what can we say about the asymptotic normality of $\widehat \theta_2$? Why?

Assuming what we showed in (a) or that $A_1$ is constant,

\[\begin{align} \sqrt{ n }(\widehat \theta_1 - \widehat \theta_2) & \stackrel{ \text{p}}{\rightarrow} b \\ \frac{ \sqrt{ n } }{ \sqrt{ A_1 } }(\widehat \theta_2 - \widehat \theta_1) & \stackrel{ \text{p}}{\rightarrow} \frac{ -b }{ \sqrt{ A_1 } } & \text{CMT}\\ \frac{ \sqrt{ n } (\widehat \theta_1 - \theta_0) }{ \sqrt{ A_1 } } + \frac{ \sqrt{ n } }{ \sqrt{ A_1 } }(\widehat \theta_2 - \widehat \theta_1) & \stackrel{ \text{d}}{\rightarrow} N(0,1) - \frac{ b }{ \sqrt{ A_1 } } & \text{Slutsky} \\ \frac{ \sqrt{ n } (\widehat \theta_2 - \theta_0) }{ \sqrt{ A_1 } }& \stackrel{ \text{d}}{\rightarrow} N\Big(- \frac{ b }{ \sqrt{ A_1 } } ,1\Big) \\ \widehat \theta_2 & \sim AN\Big( \theta_0 - \frac{ b }{ \sqrt{ n } }, \frac{ A_1 }{ n } \Big). \end{align}\]

5.24

When two independent binomials $X_1$ is $Binomial(n_1, p_1)$ and $X_2$ is $Binomial(n_2, p_2)$, are put in the form of a $2 \times 2$ table (see Example 5.31, p. 240), then one often estimates the odds ratio

\[\theta = \frac{ \frac{ p_1 }{ 1-p_1 } }{ \frac{ p_2 }{ 1-p_2 } } = \frac{ p_1 ( 1- p_2) }{ p_2 (1 - p_1) }.\]

The estimate $\widehat \theta$ is obtained by inserting $\widehat p_1 = X_1 / n_1$ and $\widehat p_2 = X_2 / n_2$ in the above expression. Show that the $\log(\widehat \theta)$ has asymptotic variance

\[\frac{ 1 }{ n_1 p_1 (1-p_1) } + \frac{ 1 }{ n_2 p_2(1-p_2) }\]

We know

\[\widehat p_1 \sim AN \Big( p_1, \frac{ p_1(1-p_1) }{ n_1 } \Big) \perp \widehat p_2 \sim AN \Big( p_2, \frac{ p_2(1-p_2) }{ n_1 } \Big).\]

Thus,

\[\begin{bmatrix} \widehat p_1 \\ \widehat p_2 \end{bmatrix} \sim AN \Big( \begin{bmatrix} p_1 \\ p_2 \end{bmatrix}, \begin{bmatrix} \frac{ p_1(1-p_1) }{ n_1 } & 0 \\ 0 & \frac{ p_2(1-p_2) }{ n_2 } \end{bmatrix} \Big).\]

Take

\(\begin{align} g(\begin{bmatrix}a & b \end{bmatrix}) & = \log(\frac{ a(1-b) }{ b(1-a) }) \\ g'(\begin{bmatrix} a & b \end{bmatrix}) & = \begin{bmatrix} \frac{ 1 }{ a(1-a) } & \frac{ 1 }{ b(b-1) } \end{bmatrix}. \end{align}\)

Then, by the delta method

\[AVar(g(\begin{bmatrix} \widehat p_1 & \widehat p_2 \end{bmatrix})) = \begin{bmatrix} \frac{ 1 }{ p_1(1-p_1) } & \frac{ 1 }{ p_2(p_2-1) } \end{bmatrix} \begin{bmatrix} \frac{ p_1(1-p_1) }{ n_1 } & 0 \\ 0 & \frac{ p_2(1-p_2) }{ n_2 } \end{bmatrix} \begin{bmatrix} \frac{ 1 }{ p_1(1-p_1) } \\ \frac{ 1 }{ p_2(p_2-1) } \end{bmatrix} = \frac{ 1 }{ n_1 p_1 (1-p_1) } + \frac{ 1 }{ n_2 p_2(1-p_2) }\]